K
Khách

Hãy nhập câu hỏi của bạn vào đây, nếu là tài khoản VIP, bạn sẽ được ưu tiên trả lời.

6 tháng 3 2016

tớ ko bít. Giúp với. Nhé

6 tháng 3 2016

Áp dụng bất đẳng thức:\(\frac{1}{x}+\frac{1}{y}+\frac{1}{z}\ge\frac{9}{x+y+z}\)

ta có:

\(A=\frac{1}{\left(a-b\right)^2}+\frac{1}{\left(b-c\right)^2}+\frac{1}{\left(c-a\right)^2}\ge\frac{9}{\left(a-b\right)^2+\left(b-c\right)^2+\left(c-a\right)^2}\)

Đến đâu Cm dưới mẫu <4 nữa là đc

Tích nha

20 tháng 3 2022

Bất đẳng thức sai, chẳng hạn với \(a=b=10^{-4};c=0,5-a-b\).

27 tháng 12 2015

ai bố thí tui 2 tick lên bảng xếp hạng đi

20 tháng 2 2021

Đặt\(P=\dfrac{a}{1+b^2}+\dfrac{b}{1+c^2}+\dfrac{c}{1+a^2+}+\dfrac{1}{2}\left(ab+bc+ca\right)\) 

Bổ đề: \(\left(a+b+c\right)\left(\dfrac{1}{a}+\dfrac{1}{b}+\dfrac{1}{c}\right)\ge9\) \(\Rightarrow\dfrac{1}{a}+\dfrac{1}{b}+\dfrac{1}{c}\ge\dfrac{9}{a+b+c}\) (1)

Chứng minh bổ đề: \(\left(a+b+c\right)\left(\dfrac{1}{a}+\dfrac{1}{b}+\dfrac{1}{c}\right)\ge9\sqrt[3]{abc.\dfrac{1}{abc}}=9\left(\forall a,b,c\ge0\right)\) 

Kết hợp điều kiện đề bài ta được: \(a+b+c\ge3\)

Ta có: \(\dfrac{ab^2}{1+b^2}\le\dfrac{ab^2}{2\sqrt{b^2}}=\dfrac{ab}{2}\) ( AM-GM cho 2 số không âm 1 và b^2 )

\(\Rightarrow\dfrac{a}{1+b^2}=a-\dfrac{ab^2}{1+b^2}\ge a-\dfrac{ab}{2}\left(1\right)\)

Chứng minh hoàn toàn tương tự: \(\dfrac{b}{1+c^2}\ge b-\dfrac{bc}{2}\left(2\right)\)

\(\dfrac{c}{1+a^2}\ge c-\dfrac{ca}{2}\left(3\right)\)

Cộng (1),(2),(3) vế theo vế thu được: \(P\ge a+b+c=3\)

Dấu "=" xảy ra tại a=b=c=1

 

20 tháng 2 2021

Cách gundefinediải của

11 tháng 4 2017

\(\frac{a^3}{\left(1+b\right)\left(1+c\right)}+\frac{b^3}{\left(1+c\right)\left(1+a\right)}+\frac{c^3}{\left(1+a\right)\left(1+b\right)}\)

Ta có:

\(\frac{a^3}{\left(1+b\right)\left(1+c\right)}+\frac{1+b}{8}+\frac{1+c}{8}\ge\frac{3a}{4}\)

\(\Leftrightarrow\frac{a^3}{\left(1+b\right)\left(1+c\right)}\ge\frac{6a-b-c-2}{8}\)

Tương tự ta có: \(\hept{\begin{cases}\frac{b^3}{\left(1+c\right)\left(1+a\right)}\ge\frac{6b-c-a-2}{8}\\\frac{c^3}{\left(1+a\right)\left(1+b\right)}\ge\frac{6c-a-b-2}{8}\end{cases}}\)

Cộng vế theo vế ta được

\(\frac{a^3}{\left(1+b\right)\left(1+c\right)}+\frac{b^3}{\left(1+c\right)\left(1+a\right)}+\frac{c^3}{\left(1+a\right)\left(1+b\right)}\ge\frac{6a-b-c-2}{8}+\frac{6b-c-a-2}{8}+\frac{6c-a-b-2}{8}\)

\(=\frac{a+b+c}{2}-\frac{3}{4}\ge\frac{3}{2}.\sqrt[3]{abc}-\frac{3}{4}=\frac{3}{2}-\frac{3}{4}=\frac{3}{4}\)

10 tháng 4 2017

Mai mình làm cho